Diễn Đàn MathScopeDiễn Đàn MathScope
  Diễn Đàn MathScope
Ghi Danh Hỏi/Ðáp Thành Viên Social Groups Lịch Ðánh Dấu Ðã Ðọc

Go Back   Diễn Đàn MathScope > Sơ Cấp > Đại Số và Lượng Giác

News & Announcements

Ngoài một số quy định đã được nêu trong phần Quy định của Ghi Danh , mọi người tranh thủ bỏ ra 5 phút để đọc thêm một số Quy định sau để khỏi bị treo nick ở MathScope nhé !

* Nội quy MathScope.Org

* Một số quy định chung !

* Quy định về việc viết bài trong diễn đàn MathScope

* Nếu bạn muốn gia nhập đội ngũ BQT thì vui lòng tham gia tại đây

* Những câu hỏi thường gặp

* Về việc viết bài trong Box Đại học và Sau đại học


Trả lời Gởi Ðề Tài Mới
 
Ðiều Chỉnh Xếp Bài
Old 26-07-2012, 09:47 AM   #31
vjpd3pz41iuai
+Thành Viên+
 
vjpd3pz41iuai's Avatar
 
Tham gia ngày: Dec 2011
Bài gởi: 303
Thanks: 129
Thanked 130 Times in 81 Posts
Trích:
Nguyên văn bởi hoangduyenkhtn View Post
Mình muốn trao đổi thêm một chút về bài toán 10. Bài toán thực chất là bài toán sau. Để thấy điều này bạn chia cả 2 vế cho abc
Cho a,b,c là các số thực dương, m là số thực không âm. Khi đó ta có:
$\frac{a^3}{a^2+b^2+mab}+\frac{b^3}{b^2+c^2+mbc}+
\frac{c^3}{c^2+a^2+mca}\ge\frac{a+b+c}{2+m} $
Bài toán này chứng minh bằng AM-GM ngược dấu.
Bài toán trên thực chất là $m=\frac{1}{abc} $
Bạn có thể thay đổi tùy ý m để được các bất đẳng thức khác và có thể sáng tạo ra nhiều bất đẳng thức khác cho dạng toán này và các bạn có thể tham khảo thêm ở các bài giảng về bất đẳng thức côsi của thầy nguyễn vũ lương.Chúc toàn thể diễn đàn ngày mới vui vẻ và làm việc hiệu quả.
Chuẩn rồi.Mình cũng từ bài toán tổng quát để đổi biến mà
[RIGHT][I][B]Nguồn: MathScope.ORG[/B][/I][/RIGHT]
 
__________________
vjpd3pz41iuai is offline   Trả Lời Với Trích Dẫn
Old 27-07-2012, 05:28 PM   #32
quykhtn
+Thành Viên+
 
Tham gia ngày: Mar 2012
Đến từ: Cái nôi của phở
Bài gởi: 259
Thanks: 78
Thanked 697 Times in 193 Posts
Trích:
Nguyên văn bởi vjpd3pz41iuai View Post
Bài 11:Cho $ x,y,z>0 $.Tìm GTLN của biểu thức:
$$P=\dfrac{x+y}{\sqrt[3]{2x^{3}+6y^{3}+xy(y-x)}+\sqrt[3]{2y^{3}+6x^{3}+xy(x-y)}}$$

Với $ a>0 $ ta có

$$ 1728(2a^3-a^2+a+6)-(5a+19)^3=(a-1)^2(3331a+3509) \geq 0 \Rightarrow \sqrt[3]{2a^3-a^2+a+6} \geq \dfrac{5a+19}{12} $$

Thay $ a $ lần lượt bởi $ \dfrac{x}{y} $ và $ \dfrac{y}{x} $ ta có

$$ \sqrt[3]{2x^3+6y^3+xy(y-x)} \geq \dfrac{5x+19y}{12} $$

$$ \sqrt[3]{2y^3+6x^3+xy(x-y)} \geq \dfrac{5y+19x}{12} $$

Từ đó ta có $ P \leq \dfrac{x+y}{2(x+y)}=\dfrac{1}{2} $

Vậy giá trị lớn nhất của $ P $ là $ \dfrac{1}{2} $ khi $ x=y $

Trích:
Nguyên văn bởi bboy114crew View Post

Bài 12:
Cho các số thực dương $x,y,z $ thỏa mãn: $x+y+z=1$.Tìm giá trị nhỏ nhất của biểu thức:
$A=\dfrac{1}{xyz}-54(x^3+y^3+z^3)-(\dfrac{x}{yz}+\dfrac{y}{xz}+\dfrac{z}{yx})$
Sử dụng giả thiết $ x+y+z=1 $ ta có

$$ A=\dfrac{1}{xyz}-\dfrac{x}{yz}-\dfrac{y}{zx}-\dfrac{z}{xy}-54(x^3+y^3+z^3) $$

$$ =\dfrac{(x+y+z)^2-x^2-y^2-z^2}{xyz}-54(x^3+y^3+z^3) $$

$$ =2\left(\dfrac{1}{x}+\dfrac{1}{y}+\dfrac{1}{z}-27(x^3+y^3+z^3)\right) $$

Xét $ f(x,y,z)=\dfrac{1}{x}+\dfrac{1}{y}+\dfrac{1}{z}-27(x^3+y^3+z^3) $

Không giảm tổng quát giả sử $ 1>x \geq y \geq z>0 $ ,ta có

$$ f(x,y,z)-f\left( x,\dfrac{y+z}{2},\dfrac{y+z}{2}\right)=\dfrac{(y-z)^2(4-81yz(y+z)^2)}{4yz(y+z)} $$

Chú ý $ 0< 3(y+z) \leq 2(x+y+z)=2 $ nên $ 4-81yz(y+z)^2 \geq \dfrac{16-81(y+z)^4}{4} \geq 0 $

$ \Rightarrow f(x,y,z) \geq f\left( x,\dfrac{y+z}{2},\dfrac{y+z}{2}\right)=g(x)=\dfrac {1}{x}+\dfrac{4}{1-x}-27x^3-\dfrac{27(1-x)^3}{4} $

Có $ g'(x)=\dfrac{-1}{x^2}+\dfrac{4}{(1-x)^2}-81x^2+\dfrac{81(1-x)^2}{4}=\dfrac{(3x-1)^2(3x-2)(x+1)(9x-9x^2+2)}{4x^2(1-x)^2} $

Lập bảng biến thiên ta có $ g(x) \geq g\left(\dfrac{2}{3}\right)=\dfrac{21}{4} $

Vậy giá trị nhỏ nhất của biểu thức A là $ \dfrac{21}{2} $ khi $ x=\dfrac{2}{3},y=z=\dfrac{1}{6} $ và các hoán vị.

P/S: Một cách tiếp cận khác cho bài toán này có thể xem trong cuốn Sử dụng phương pháp Cauchy-Schwarz để chứng minh bất đẳng thức của hai tác giả Võ Quốc Bá Cẩn - Trần Quốc Anh.Bài toán trên tương tự bài 193 trong cuốn sách này.


Bài toán 13 :

Cho $ a,b,c $ là độ dài ba cạnh của một tam giác ( có thể suy biến ) thỏa mãn $ a+b+c=3 $.Chứng minh rằng

$$ \dfrac{1}{a}+\dfrac{1}{b}+\dfrac{1}{c}+\dfrac{19}{ 12} \geq \dfrac{6(a^4+b^4+c^4)}{7} $$

Đẳng thức xảy ra $ \Leftrightarrow a=\dfrac{3}{2} ; b=1 ; c=\dfrac{1}{2} $ và các hoán vị.
[RIGHT][I][B]Nguồn: MathScope.ORG[/B][/I][/RIGHT]
 
__________________
The love make us weaker

Autumn

thay đổi nội dung bởi: quykhtn, 27-07-2012 lúc 10:02 PM
quykhtn is offline   Trả Lời Với Trích Dẫn
The Following 9 Users Say Thank You to quykhtn For This Useful Post:
Akira Vinh HD (02-08-2012), bboy114crew (05-08-2012), K56khtn (06-08-2012), kieudinhminh (03-02-2013), lovetohop (22-08-2012), mrvui123 (21-09-2012), Trầm (05-08-2012), vanthanh0601 (05-08-2012), vjpd3pz41iuai (05-08-2012)
Old 05-08-2012, 01:45 PM   #33
VYKA
+Thành Viên+
 
Tham gia ngày: Jun 2012
Bài gởi: 9
Thanks: 8
Thanked 1 Time in 1 Post
Trích:
Nguyên văn bởi tanggo View Post
Bài 2:
Cho các số dương $a, b, c$ thỏa mãn $abc=1$. Chứng minh rằng:
$$\dfrac{1}{a^2-a+1}+\dfrac{1}{b^2-b+1}+\dfrac{1}{c^2-c+1} \le 3$$

-------------------------------------
Ta có BĐT quen thuộc:

\[\frac{{1 + {a^k}}}{{1 + {a^k} + {a^{2k}}}} + \frac{{1 + {b^k}}}{{1 + {b^k} + {b^{2k}}}} + \frac{{1 + {c^k}}}{{1 + {c^k} + {c^{2k}}}} \le 2\]

thật vậy:

\[ineq \Leftrightarrow \sum {\frac{{{a^{2k}}}}{{1 + {a^k} + {a^{2k}}}} \ge 1} \]

\[ \Leftrightarrow \sum {\frac{1}{{1 + \frac{1}{{{a^k}}} + \frac{1}{{{a^{2k}}}}}}} \ge 1\]

BĐT cuối là BĐT vacs rồi.
----------------------------

Dễ dàng ta chứng minh được BĐT sau:

\[\frac{1}{{{a^2} - a + 1}} \le \frac{3}{2}.\frac{{1 + {a^2}}}{{1 + {a^2} + {a^4}}}\]

\[ \Rightarrow LHS \le \frac{3}{2}\sum {\frac{{1 + {a^2}}}{{1 + {a^2} + {a^4}}}} \le 3\]

BĐT được CM.
[RIGHT][I][B]Nguồn: MathScope.ORG[/B][/I][/RIGHT]
 
VYKA is offline   Trả Lời Với Trích Dẫn
The Following User Says Thank You to VYKA For This Useful Post:
hoang_kkk (28-01-2013)
Old 06-08-2012, 04:59 PM   #34
quykhtn
+Thành Viên+
 
Tham gia ngày: Mar 2012
Đến từ: Cái nôi của phở
Bài gởi: 259
Thanks: 78
Thanked 697 Times in 193 Posts
Trích:
Nguyên văn bởi quykhtn View Post
Bài toán 13 :

Cho $ a,b,c $ là độ dài ba cạnh của một tam giác ( có thể suy biến ) thỏa mãn $ a+b+c=3 $.Chứng minh rằng

$$ \dfrac{1}{a}+\dfrac{1}{b}+\dfrac{1}{c}+\dfrac{19}{ 12} \geq \dfrac{6(a^4+b^4+c^4)}{7} $$

Đẳng thức xảy ra $ \Leftrightarrow a=\dfrac{3}{2} ; b=1 ; c=\dfrac{1}{2} $ và các hoán vị.


Chú ý rằng

$$ 2\left(a^2b^2+b^2c^2+c^2a^2\right)-\left(a^4+b^4+c^4\right)=(a+b+c)(b+c-a)(c+a-b)(a+b-c) \geq 0 $$
$$ \Rightarrow (a^2+b^2+c^2)^2 \geq 2(a^4+b^4+c^4) $$
Ta chỉ cần chứng minh
$$ \dfrac{1}{a}+\dfrac{1}{b}+\dfrac{1}{c}+\dfrac{19}{ 12} \geq \dfrac{3\left(a^2+b^2+c^2 \right)^2}{7} $$
Đặt $ a=y+z,b=z+x,c=x+y \Rightarrow x,y,z \geq 0; x+y+z=\dfrac{3}{2} $

Bất đẳng thức trở thành

$$ \dfrac{1}{y+z}+\dfrac{1}{z+x}+\dfrac{1}{x+y}+ \dfrac{19}{12} \geq \dfrac{12\left(x^2+y^2+z^2+xy+yz+zx\right)^2}{7} $$

Đặt $ 4(xy+yz+zx)=t \Rightarrow 3 \geq t > 0 $

Ta có $ x^2+y^2+z^2+xy+yz+zx=(x+y+z)^2-(xy+yz+zx)=\dfrac{9-t}{4} $

Nhân cả hai vế với $ x+y+z $ và chú ý rằng

$$ (x+y+z)\left(\dfrac{1}{y+z}+\dfrac{1}{z+x}+\dfrac{ 1}{x+y}\right)=3+\dfrac{x}{y+z}+\dfrac{y}{z+x}+ \dfrac{z}{x+y} \geq 3+\dfrac{x^2+y^2+z^2}{xy+yz+zx}=1+\dfrac{9}{t} $$

Ta chỉ cần chứng minh

$$ \dfrac{9}{t}+\dfrac{27}{8} \geq \dfrac{9(9-t)^2}{56} $$
$$ \Leftrightarrow t^3-18t^2+6t-56 \leq 0 $$
$$ \Leftrightarrow (t-2)^2(t-14) \leq 0 $$

Bất đẳng thức này hiển nhiên đúng.

Đẳng thức xảy ra $ \Leftrightarrow x+y+z=\dfrac{3}{2} ; xy+yz+zx=\dfrac{1}{2} ;xyz=0 \Leftrightarrow (x,y,z)=\left(0;\dfrac{1}{2};1\right)$
hay $ a=\dfrac{3}{2}, b=1 ,c=\dfrac{1}{2} $ và các hoán vị tương ứng.

[RIGHT][I][B]Nguồn: MathScope.ORG[/B][/I][/RIGHT]
 
__________________
The love make us weaker

Autumn
quykhtn is offline   Trả Lời Với Trích Dẫn
The Following 4 Users Say Thank You to quykhtn For This Useful Post:
Conanvn (06-08-2012), K56khtn (06-08-2012), lovetohop (22-08-2012), Trầm (06-08-2012)
Old 06-08-2012, 05:51 PM   #35
Chém Gió
+Thành Viên+
 
Tham gia ngày: Nov 2011
Bài gởi: 60
Thanks: 0
Thanked 28 Times in 18 Posts
Trích:
Nguyên văn bởi quykhtn View Post
Bài toán 13 :

Cho $ a,b,c $ là độ dài ba cạnh của một tam giác ( có thể suy biến ) thỏa mãn $ a+b+c=3 $.Chứng minh rằng

$$ \dfrac{1}{a}+\dfrac{1}{b}+\dfrac{1}{c}+\dfrac{19}{ 12} \geq \dfrac{6(a^4+b^4+c^4)}{7} $$

Đẳng thức xảy ra $ \Leftrightarrow a=\dfrac{3}{2} ; b=1 ; c=\dfrac{1}{2} $ và các hoán vị.
Do $a,b,c$ là độ dài ba cạnh của một tam giác (có thể suy biến) nên tồn tại các số không âm $x,y,z$ để cho
$$2a=y+z,2b=z+x,2c=x+y.$$
Khi ấy $x+y+z=3$ và bất đẳng thức của ta trở thành
$$16\sum\frac{1}{y+z}+\frac{38}{3}\ge\frac{3}{7} \sum (y+z)^4.$$
Đổi biến $xy+yz+zx=q,xyz=r,$ ta có $r\ge 0,0\le q\le 3$ và bất đẳng thức trên tương đương với
$$\frac{8(9+q)}{3q-r}+\frac{19}{3}\ge\frac{3}{7}(81-18q+q^2-12r).$$
Vì $r\ge 0$ nên ta chỉ cần chứng minh
$$\frac{8(9+q)}{3q}+\frac{19}{3}\ge\frac{3}{7}(81-18q+q^2).$$
Bất đẳng thức này tương đương với bất đẳng thức hiển nhiên đúng
$$\frac{3(2-q)^2(14-q)}{7q}\ge0.$$
[RIGHT][I][B]Nguồn: MathScope.ORG[/B][/I][/RIGHT]
 
Chém Gió is offline   Trả Lời Với Trích Dẫn
Old 07-08-2012, 12:11 PM   #36
quykhtn
+Thành Viên+
 
Tham gia ngày: Mar 2012
Đến từ: Cái nôi của phở
Bài gởi: 259
Thanks: 78
Thanked 697 Times in 193 Posts
Bài toán 14 .Cho các số thực dương $ a,b,c $.Chứng minh rằng

$$ \left(\dfrac{a^2}{b^2+c^2}+\dfrac{1}{3} \right)\left(\dfrac{b^2}{c^2+a^2}+\dfrac{1}{3} \right)\left(\dfrac{c^2}{a^2+b^2}+\dfrac{1}{3} \right) \geq \left(\dfrac{a}{b+c}+\dfrac{1}{3}\right) \left(\dfrac{b}{c+a}+\dfrac{1}{3} \right)\left(\dfrac{c}{a+b}+\dfrac{1}{3} \right). $$
[RIGHT][I][B]Nguồn: MathScope.ORG[/B][/I][/RIGHT]
 
__________________
The love make us weaker

Autumn
quykhtn is offline   Trả Lời Với Trích Dẫn
The Following 2 Users Say Thank You to quykhtn For This Useful Post:
K56khtn (08-08-2012), Trầm (07-08-2012)
Old 09-08-2012, 08:49 PM   #37
Trầm
+Thành Viên Danh Dự+
 
Tham gia ngày: Feb 2011
Bài gởi: 657
Thanks: 388
Thanked 470 Times in 196 Posts
Lấy logarit nepe hai vế, ta được bất đẳng thức:
$$\ln\left(\dfrac{a^2}{b^2+c^2}+\dfrac{1}{3}\right )
+\ln\left(\dfrac{b^2}{c^2+a^2}+\dfrac{1}{3}\right)
+\ln\left(\dfrac{c^2}{a^2+b^2}+\dfrac{1}{3}\right)
\ge \ln\left(\dfrac{a}{b+c}+\dfrac{1}{3}\right)
+\ln\left(\dfrac{b}{c+a}+\dfrac{1}{3}\right)
+\ln\left(\dfrac{c}{a+b}+\dfrac{1}{3}\right)$$
Như vậy ta sẽ chứng minh hàm số:
$f(x)=\ln\left(\dfrac{a^x}{b^x+c^x}
+\dfrac{1}{3}\right)
+\ln\left(\dfrac{b^x}{c^x+a^x}
+\dfrac{1}{3}\right)
+\ln\left(\dfrac{c^x}{a^x+b^x}
+\dfrac{1}{3}\right)$ đồng biến với $a, b, c, x >0$


Ta có:
$$f'(x)=\sum \dfrac{3a^x\left[\ln a(b^x+c^x)-\ln b.b^x-\ln c.c^x\right]}{(b^x+c^x)(3a^x+b^x+c^x)}
=\sum \dfrac{3a^xb^x(a^x-b^x)(\ln a - \ln b)(a^x+b^x-c^x)}{(3a^x+b^x+c^x)(3b^x+c^x+a^x)(b^x+c^x)(a^x+c^ x)}$$

Không mất tính tổng quát, giả sử $a\ge b \ge c >0$.
Ta có:
$(a^x-c^x)(\ln a - \ln c)= (a^x-b^x+b^x-c^x)(\ln a - \ln b + \ln b - \ln c)$
$=(a^x-b^x)(\ln a-\ln b)+(a^x-b^x)(\ln b - \ln c)+(b^x-c^x)(\ln a - \ln b)+(b^x-c^x)(\ln b - \ln c) \ge (b^x-c^x)(\ln b - \ln c)$


Do đó:
$f'(x) \ge \dfrac{3b^xc^x(b^x-c^x)(\ln b - \ln c)(c^x+b^x-a^x)}{(3b^x+c^x+a^x)(3c^x+a^x+b^x)(c^x+a^x)(a^x+b^ x)}
+\dfrac{3c^xa^x(b^x-c^x)(\ln b - \ln c)(c^x+a^x-b^x)}{(3c^x+a^x+b^x)(3a^x+b^x+c^x)(a^x+b^x)(b^x+c^ x)}$
$=\dfrac{3c^x(b^x-c^x)(\ln b - \ln c)\left[b^x(c^x+b^x-a^x)(b^x+c^x)(3a^x+b^x+c^x)+a^x(c^x+a^x-b^x)(3b^x+a^x+c^x)(a^x+b^x)\right]}{(a^x+b^x)(b^x+c^x)(c^x+a^x)(3a^x+b^x+c^x)(3b^x+c ^x+a^x)(3c^x+a^x+b^x)}$


Đặt $x=a^x,y=b^x,z=c^x$.
Ta có:
$y(y+z-x)(3x+y+z)(y+z)+x(z+x-y)(3y+z+x)(z+x) \ge y(y+z-x)(3x+y+z)(y+z)+y(z+x-y)(3y+z+x)(y+z)>0$
Do đó $f'(x) \ge 0 \forall x >0$.
$f'(x)=0 \Leftrightarrow a= b=c$
Vậy hàm số đồng biến và bất đẳng thức được chứng minh.
[RIGHT][I][B]Nguồn: MathScope.ORG[/B][/I][/RIGHT]
 
__________________

thay đổi nội dung bởi: Trầm, 09-08-2012 lúc 09:50 PM
Trầm is offline   Trả Lời Với Trích Dẫn
The Following 4 Users Say Thank You to Trầm For This Useful Post:
K56khtn (11-08-2012), lovetohop (22-08-2012), quykhtn (09-08-2012), zớt (13-08-2012)
Old 09-08-2012, 09:42 PM   #38
quykhtn
+Thành Viên+
 
Tham gia ngày: Mar 2012
Đến từ: Cái nôi của phở
Bài gởi: 259
Thanks: 78
Thanked 697 Times in 193 Posts
tanggo nên dùng kí hiệu $ \sum $ để lời giải gọn hơn

Bài toán 15 Cho các số thực dương $ a,b,c $ thỏa mãn $ abc=1 $.Chứng minh rằng $$ 1) \ \ \ \ \sqrt[3]{\dfrac{a^3+b^3}{2}}+\sqrt[3]{\dfrac{b^3+c^3}{2}}+\sqrt[3]{\dfrac{c^3+a^3}{2}}+6 \leq 3(a+b+c) $$ $$ 2) \ \ \ \ \sqrt[4]{\dfrac{a^4+b^4}{2}}+\sqrt[4]{\dfrac{b^4+c^4}{2}}+\sqrt[4]{\dfrac{c^4+a^4}{2}}+\dfrac{21}{2} \leq \dfrac{9(a+b+c)}{2} $$
[RIGHT][I][B]Nguồn: MathScope.ORG[/B][/I][/RIGHT]
 
__________________
The love make us weaker

Autumn
quykhtn is offline   Trả Lời Với Trích Dẫn
The Following 2 Users Say Thank You to quykhtn For This Useful Post:
K56khtn (11-08-2012), Trầm (09-08-2012)
Old 13-08-2012, 01:47 PM   #39
Snow Bell
+Thành Viên Danh Dự+
 
Tham gia ngày: Apr 2012
Đến từ: Heaven
Bài gởi: 579
Thanks: 10
Thanked 513 Times in 283 Posts
Trích:
Nguyên văn bởi quykhtn View Post
tanggo nên dùng kí hiệu $ \sum $ để lời giải gọn hơn

Bài toán 15 Cho các số thực dương $ a,b,c $ thỏa mãn $ abc=1 $.Chứng minh rằng $$ 1) \ \ \ \ \sqrt[3]{\dfrac{a^3+b^3}{2}}+\sqrt[3]{\dfrac{b^3+c^3}{2}}+\sqrt[3]{\dfrac{c^3+a^3}{2}}+6 \leq 3(a+b+c) $$
Bài toán này có thể chứng minh bằng cách sử dụng đánh giá sau:
$$ \sqrt[3]{\frac{a^3+b^3}{2}} \le \frac{a^2+b^2}{a+b} $$
Chú ý rằng:
$$ \frac{a^2+b^2}{a+b}=a+b-\frac{2ab}{a+b} $$
Như vậy ta phải chứng minh:
$$ 2\left[\frac{ab}{a+b}+\frac{bc}{b+c}+\frac{ca}{c+a} \right]+a+b+c \ge 6 $$
Áp dụng bất đẳng thức AM-GM với giả thiết $ abc=1 $,ta có ngay:
$$ \frac{2ab}{a+b}+\frac{a+b}{2}+ \frac{2bc}{b+c}+ \frac{b+c}{2}+\frac{2ca}{c+a}+\frac{c+a}{2} \ge 6 $$
Vậy ta có ìiều phải chứng minh.Đẳng thức xảy ra khi $ a=b=c=1 $
[RIGHT][I][B]Nguồn: MathScope.ORG[/B][/I][/RIGHT]
 
Snow Bell is offline   Trả Lời Với Trích Dẫn
The Following 2 Users Say Thank You to Snow Bell For This Useful Post:
K56khtn (16-08-2012), quykhtn (22-08-2012)
Old 16-08-2012, 06:56 PM   #40
K56khtn
+Thành Viên+
 
Tham gia ngày: Mar 2012
Đến từ: Hà Nội
Bài gởi: 343
Thanks: 244
Thanked 285 Times in 177 Posts
Trích:
Nguyên văn bởi quykhtn View Post
tanggo nên dùng kí hiệu $ \sum $ để lời giải gọn hơn

Bài toán 15 Cho các số thực dương $ a,b,c $ thỏa mãn $ abc=1 $.Chứng minh rằng
$$ 2) \ \ \ \ \sqrt[4]{\dfrac{a^4+b^4}{2}}+\sqrt[4]{\dfrac{b^4+c^4}{2}}+\sqrt[4]{\dfrac{c^4+a^4}{2}}+\dfrac{21}{2} \leq \dfrac{9(a+b+c)}{2} $$
Tiếp tục topic nào

Ý tưởng lời giải $ 2) $ chắc là tương tự $ 1) $ tức là tìm $ k $ sao cho

$$ \sqrt[4]{\dfrac{a^4+b^4}{2}} \leq \dfrac{2(a^2+b^2-kab)}{(2-k)(a+b)}$$

Với $ 0 \leq k <2 $

Vĩnh Phúc thử đi theo hướng này xem
[RIGHT][I][B]Nguồn: MathScope.ORG[/B][/I][/RIGHT]
 
__________________
Nguyễn Ngọc Khanh

thay đổi nội dung bởi: Trầm, 16-08-2012 lúc 09:41 PM
K56khtn is offline   Trả Lời Với Trích Dẫn
Old 16-08-2012, 08:46 PM   #41
hoduckhanhgx
+Thành Viên+
 
hoduckhanhgx's Avatar
 
Tham gia ngày: Dec 2010
Bài gởi: 40
Thanks: 138
Thanked 45 Times in 15 Posts
Trích:
Nguyên văn bởi hoangduyenkhtn View Post
Mình muốn trao đổi thêm một chút về bài toán 10. Bài toán thực chất là bài toán sau. Để thấy điều này bạn chia cả 2 vế cho abc
Cho a,b,c là các số thực dương, m là số thực không âm. Khi đó ta có:
$\frac{a^3}{a^2+b^2+mab}+\frac{b^3}{b^2+c^2+mbc}+
\frac{c^3}{c^2+a^2+mca}\ge\frac{a+b+c}{2+m} $
Bài toán này chứng minh bằng AM-GM ngược dấu.
Bài toán trên thực chất là $m=\frac{1}{abc} $
Bạn có thể thay đổi tùy ý m để được các bất đẳng thức khác và có thể sáng tạo ra nhiều bất đẳng thức khác cho dạng toán này và các bạn có thể tham khảo thêm ở các bài giảng về bất đẳng thức côsi của thầy nguyễn vũ lương.Chúc toàn thể diễn đàn ngày mới vui vẻ và làm việc hiệu quả.
Đây là một phần trong bài viết về vấn đề này của mình và mình có lí do riêng nên không thể chia sẻ hết được, viết cũng khá lâu rồi đầu năm lớp 10. Bây giờ nhìn lại thấy xoàng quá

[RIGHT][I][B]Nguồn: MathScope.ORG[/B][/I][/RIGHT]
 
File Kèm Theo
Kiểu File : doc Hãy nhìn một bài toán từ nhiều hướng.doc (175.0 KB, 112 lần tải)
hoduckhanhgx is offline   Trả Lời Với Trích Dẫn
The Following User Says Thank You to hoduckhanhgx For This Useful Post:
SthgQuynh (23-11-2012)
Old 22-08-2012, 12:05 PM   #42
quykhtn
+Thành Viên+
 
Tham gia ngày: Mar 2012
Đến từ: Cái nôi của phở
Bài gởi: 259
Thanks: 78
Thanked 697 Times in 193 Posts
Trích:
Nguyên văn bởi quykhtn View Post

Bài toán 15 Cho các số thực dương $ a,b,c $ thỏa mãn $ abc=1 $.Chứng minh rằng
$$ 2) \ \ \ \ \sqrt[4]{\dfrac{a^4+b^4}{2}}+\sqrt[4]{\dfrac{b^4+c^4}{2}}+\sqrt[4]{\dfrac{c^4+a^4}{2}}+\dfrac{21}{2} \leq \dfrac{9(a+b+c)}{2} $$

Sử dụng bất đẳng thức AM-GM ta có

$$ \dfrac{a+b}{8}+\dfrac{a+b}{8}+\dfrac{a+b}{8}+ \dfrac{a^4+b^4}{(a+b)^3} \geq \sqrt[4]{\dfrac{a^4+b^4}{2}} $$

Mặt khác $ a^4+b^4=(a+b)^4-2ab(2a^2+3ab+2b^2) \leq (a+b)^4-\dfrac{7ab(a+b)^2}{2} $

Từ đó ta có
$$ \sqrt[4]{\dfrac{a^4+b^4}{2}} \leq \dfrac{11(a+b)}{8}-\dfrac{7ab}{2(a+b)} $$

Xây dựng hai bất đẳng thức tương tự ta chỉ cần chứng minh

$$ \dfrac{11(a+b+c)}{4}-\dfrac{7ab}{2(a+b)}-\dfrac{7bc}{2(b+c)}-\dfrac{7ca}{2(c+a)}+\dfrac{21}{2} \leq \dfrac{9(a+b+c)}{2} $$

$$ \Leftrightarrow 6 \leq \dfrac{a+b}{2}+\dfrac{b+c}{2}+\dfrac{c+a}{2}+ \dfrac{2ab}{a+b}+ \dfrac{2bc}{b+c}+\dfrac{2ca}{c+a} $$

Bất đẳng thức cuối đúng theo bất đẳng thức AM-GM với chú ý $ abc=1 $

Đẳng thức xảy ra $ \Leftrightarrow a=b=c=1 $.

Tiếp tục topic nào các bạn !
[RIGHT][I][B]Nguồn: MathScope.ORG[/B][/I][/RIGHT]
 
__________________
The love make us weaker

Autumn
quykhtn is offline   Trả Lời Với Trích Dẫn
The Following 3 Users Say Thank You to quykhtn For This Useful Post:
dung_toan78 (22-08-2012), K56khtn (23-08-2012), lovetohop (22-08-2012)
Old 22-08-2012, 06:49 PM   #43
tffloorz
+Thành Viên+
 
Tham gia ngày: Nov 2011
Bài gởi: 140
Thanks: 140
Thanked 24 Times in 20 Posts
Bài 16: Cho $ a , b , c \ge 0 $ và không có hai số nào đồng thời bằng $0$. Chứng minh rằng:
$$ (ab + bc +ca)[\frac{1}{(b + c)^2} + \frac{1}{(c + a)^2} + \frac{1}{(a + b)^2}] \ge \frac{9}{4} $$
[RIGHT][I][B]Nguồn: MathScope.ORG[/B][/I][/RIGHT]
 

thay đổi nội dung bởi: Trầm, 22-08-2012 lúc 06:59 PM
tffloorz is offline   Trả Lời Với Trích Dẫn
Old 23-08-2012, 01:53 PM   #44
K56khtn
+Thành Viên+
 
Tham gia ngày: Mar 2012
Đến từ: Hà Nội
Bài gởi: 343
Thanks: 244
Thanked 285 Times in 177 Posts
Trích:
Nguyên văn bởi tffloorz View Post
Bài 16: Cho $ a , b , c \ge 0 $ và không có hai số nào đồng thời bằng $0$. Chứng minh rằng:
$$ (ab + bc +ca)[\frac{1}{(b + c)^2} + \frac{1}{(c + a)^2} + \frac{1}{(a + b)^2}] \ge \frac{9}{4} $$
Có thể xem lời giải bài toán này ở đây
[Only registered and activated users can see links. ]
,đây là bất đẳng thức Iran 96 quen thuộc đã thảo luận nhiều trên diễn đàn rồi
[RIGHT][I][B]Nguồn: MathScope.ORG[/B][/I][/RIGHT]
 
__________________
Nguyễn Ngọc Khanh
K56khtn is offline   Trả Lời Với Trích Dẫn
Old 23-08-2012, 01:55 PM   #45
ratuno
+Thành Viên+
 
Tham gia ngày: Jun 2012
Bài gởi: 17
Thanks: 3
Thanked 8 Times in 6 Posts
Trích:
Nguyên văn bởi tffloorz View Post
Bài 16: Cho $ a , b , c \ge 0 $ và không có hai số nào đồng thời bằng $0$. Chứng minh rằng:
$$ (ab + bc +ca)[\frac{1}{(b + c)^2} + \frac{1}{(c + a)^2} + \frac{1}{(a + b)^2}] \ge \frac{9}{4} $$
Lời giải trong một cuốn sách BĐT như sau. Đặt $p=a+b+c$, $q=ab+bc+ca$, $r=abc$. BĐT đã cho sẽ tương đương với $3pq(p^3 −4pq +9r)+ q(p^4 −5p^2q +4q^2 +6pr)+r(pq −9r) \ge 0$
[RIGHT][I][B]Nguồn: MathScope.ORG[/B][/I][/RIGHT]
 
ratuno is offline   Trả Lời Với Trích Dẫn
Trả lời Gởi Ðề Tài Mới

Bookmarks

Ðiều Chỉnh
Xếp Bài

Quuyền Hạn Của Bạn
You may not post new threads
You may not post replies
You may not post attachments
You may not edit your posts

BB code is Mở
Smilies đang Mở
[IMG] đang Mở
HTML đang Tắt

Chuyển đến


Múi giờ GMT. Hiện tại là 04:02 PM.


Powered by: vBulletin Copyright ©2000-2024, Jelsoft Enterprises Ltd.
Inactive Reminders By mathscope.org
[page compression: 116.07 k/132.55 k (12.43%)]